LSAT and Law School Admissions Forum

Get expert LSAT preparation and law school admissions advice from PowerScore Test Preparation.

 Administrator
PowerScore Staff
  • PowerScore Staff
  • Posts: 8950
  • Joined: Feb 02, 2011
|
#33682
Complete Question Explanation

Flaw in the Reasoning. The correct answer choice is (A)

In this stimulus, the author starts off with a version of the “some people say...” rhetorical device. Typically, an author will introduce some other party’s viewpoint and then will immediately conclude that it is incorrect. Here the application of that technique is a bit different. Rather than talking about what some other person thinks, the author tells us what some other people do. Here, some video game makers sell the movie rights for their popular games, which the author concludes is “rarely good from a business perspective.” In support of that conclusion, the author offers the example of video game maker StarQuanta, which sold the movie rights to its game Nostroma. Although Nostroma was a popular game, the movie adaptation was not. In fact, critics and fans hated the movie, and subsequent better made versions of the Nostroma game sold poorly.

This argument is flawed, because it reaches a general conclusion, that selling the movie rights to a popular game is rarely good, based on just one example, that of the Nostroma movie adaptation debacle. While it may be the case that StarQuanta’s experience with Nostroma is typical of what occurs when game makers sell off their film rights, we do not have any reason to think that is the case. For all we know, StarQuanta had a uniquely poor experience.

The question stem identifies this as a Flaw in the Reasoning question. Our prephrase is that the correct answer choice will describe the author’s error in basing a general conclusion on a single, potentially atypical example.

Answer choice (A): This is the correct answer choice, and it closely follows our prephrase.

Answer choice (B): Here, the answer choice is inconsistent with the stimulus. The author’s inference is that it is not a good business idea to sell the movie rights to a popular video game, not that something will be disliked by the public solely because the critics did not like it.

Answer choice (C): This answer choice describes a circular argument. However, the argument was not circular because the evidence, about StarQuanta’s experience, was not the same as the generalized conclusion.

Answer choice (D): As with answer choice (B), this answer choice is inconsistent with the stimulus, since the author does not assume that the movie adaptation of the game will be as popular as the game itself.

Answer choice (E): This answer choice is incorrect because it describes a Mistaken Reversal, which is a conditional reasoning error. However, the conclusion did not result from an application of conditional reasoning.
 stephodigie
  • Posts: 5
  • Joined: Jul 16, 2018
|
#59016
Could you explain in more detail why C is not the right answer? I am still a little confused on that.
 Rachael Wilkenfeld
PowerScore Staff
  • PowerScore Staff
  • Posts: 1419
  • Joined: Dec 15, 2011
|
#59146
Hi Steph,

Answer choice (C) describes a circular scenario---where the argument is comprised of a conclusion that is merely a restatement of the premise(s). Here, our conclusion is that selling the movie rights for popular games rarely makes sense. But the premises are more than a simple restatement of the conclusion. The premises contain an example to show a situation where it did not make sense to sell the movie rights.

Circular arguments are different. There won't be additional evidence presented like in this stimulus. An example of a circular argument would be: "This book is true because it says so in the book." We don't have any evidence the book is true other than the book. Circular arguments are not super common on the LSAT in flaw questions because they are so unusual in structure. But they are fairly common INCORRECT answer choices.

Hope that helps!
Rachael
User avatar
 Albertlyu
  • Posts: 98
  • Joined: Jul 18, 2020
|
#79775
hi, may I ask why D is incorrect?

from the stimulus, we know that this film adaption of the game N was poorly made, but that does not invariably have to be the case, therefore by saying that this "move" is rarely right, the author assumes that every film adapted from games will be badly made.

thanks

Albert
 Jeremy Press
PowerScore Staff
  • PowerScore Staff
  • Posts: 1000
  • Joined: Jun 12, 2017
|
#80023
Hi Albert,

The problem is that your understanding of the assumption (which is okay!) doesn't match with what's stated in answer choice D. Although the author assumes that movies adapted from games will be unpopular, the game in question was actually popular. So the game and the movie were not "of roughly equal popularity." And since we don't know whether the author thinks that other similar games will be popular or unpopular (all we have to go on is that the author probably thinks they'll mirror Nostroma, and be popular), we have no basis for saying the author assumes the games and movies will be equal in their popularity.

I hope this helps!
User avatar
 Albertlyu
  • Posts: 98
  • Joined: Jul 18, 2020
|
#82921
Jeremy Press wrote: Wed Oct 14, 2020 3:52 pm Hi Albert,

The problem is that your understanding of the assumption (which is okay!) doesn't match with what's stated in answer choice D. Although the author assumes that movies adapted from games will be unpopular, the game in question was actually popular. So the game and the movie were not "of roughly equal popularity." And since we don't know whether the author thinks that other similar games will be popular or unpopular (all we have to go on is that the author probably thinks they'll mirror Nostroma, and be popular), we have no basis for saying the author assumes the games and movies will be equal in their popularity.

I hope this helps!
it does, thank you, Jeremy!

Get the most out of your LSAT Prep Plus subscription.

Analyze and track your performance with our Testing and Analytics Package.